Difference between revisions of "2025 AIME II Problems/Problem 4"
(→Solution 1) |
(→Solution 2) |
||
Line 42: | Line 42: | ||
\frac{15}{12} \cdot \frac{24}{21} \cdot \frac{35}{32} \cdots = \frac{3\cdot 5}{2\cdot 6} \cdot \frac{4\cdot 6}{3\cdot 7} \cdot \frac{5\cdot 7}{4\cdot 8} \cdots \frac{62\cdot 64}{61\cdot 65} = \frac{5}{2} \cdot \frac{62}{65} = \frac{31}{13} | \frac{15}{12} \cdot \frac{24}{21} \cdot \frac{35}{32} \cdots = \frac{3\cdot 5}{2\cdot 6} \cdot \frac{4\cdot 6}{3\cdot 7} \cdot \frac{5\cdot 7}{4\cdot 8} \cdots \frac{62\cdot 64}{61\cdot 65} = \frac{5}{2} \cdot \frac{62}{65} = \frac{31}{13} | ||
\end{align*} | \end{align*} | ||
− | Multiplying these together gets us the original product, which is <math>\frac{31}{13} \cdot 3 = \frac{93}{13}</math>. | + | Multiplying these together gets us the original product, which is <math>\frac{31}{13} \cdot 3 = \frac{93}{13}</math>. |
Thus <math>m+n=\boxed{106}</math>. | Thus <math>m+n=\boxed{106}</math>. | ||
+ | |||
+ | ~ Edited by [https://artofproblemsolving.com/wiki/index.php/User:Aoum aoum] | ||
== Solution 3 == | == Solution 3 == |
Revision as of 16:19, 14 February 2025
Problem
The productis equal to
where
and
are relatively prime positive integers. Find
Solution 1
We can rewrite the equation as:
\begin{align*} &= \dfrac{15}{12} \cdot \dfrac{24}{21} \cdot \dfrac{35}{32} \cdot \dots \cdot \dfrac{3968}{3965} \cdot \dfrac{\log_4{5}}{\log_{64}{5}} \\ &= \log_4{64} \cdot \dfrac{(4+1)(4-1)(5+1)(5-1)\cdot \dots \cdot (63+1)(63-1)}{(4+2)(4-2)(5+2)(5-2)\cdot \dots \cdot (63+2)(63-2)} \\ &= 3 \cdot \dfrac{5 \cdot 3 \cdot 6 \cdot 4 \cdot \dots \cdot 64 \cdot 62}{6 \cdot 2 \cdot 7 \cdot 3 \cdot \dots \cdot 65 \cdot 61} \\ &= 3 \cdot \dfrac{5 \cdot 62}{65 \cdot 2} \\ &= 3 \cdot \dfrac{5 \cdot 2 \cdot 31}{5 \cdot 13 \cdot 2} \\ &= 3 \cdot \dfrac{31}{13} \\ &= \dfrac{93}{13} \end{align*}
Desired answer:
(Feel free to correct any and formatting.)
~ Mitsuihisashi14
~ by Tacos_are_yummy_1
~ Additional edits by aoum
Solution 2
We can move the exponents to the front of the logarithms like this:
\begin{align*}
\frac{\log_4 (5^{15})}{\log_5 (5^{12})} \cdot \frac{\log_5 (5^{24})}{\log_6 (5^{21})}\cdot \frac{\log_6 (5^{35})}{\log_7 (5^{32})} \cdots = \frac{15\log_4 (5)}{12\log_5 (5)} \cdot \frac{24\log_5 (5)}{21\log_6 (5)}\cdot \frac{35\log_6 (5)}{32\log_7 (5)} \cdots
\end{align*}
Now we multiply the logs and fractions seperately.\\
Let's do it for the logs first:
\begin{align*}
\frac{\log_4 (5)}{\log_5 (5)} \cdot \frac{\log_5 (5)}{\log_6 (5)}\cdot \frac{\log_6 (5)}{\log_7 (5)} \cdots \frac{\log_{63} (5)}{\log_{64} (5)} = \frac{\log_4 (5)}{\log_{64} (5)} = 3
\end{align*}
Now fractions:
\begin{align*}
\frac{15}{12} \cdot \frac{24}{21} \cdot \frac{35}{32} \cdots = \frac{3\cdot 5}{2\cdot 6} \cdot \frac{4\cdot 6}{3\cdot 7} \cdot \frac{5\cdot 7}{4\cdot 8} \cdots \frac{62\cdot 64}{61\cdot 65} = \frac{5}{2} \cdot \frac{62}{65} = \frac{31}{13}
\end{align*}
Multiplying these together gets us the original product, which is .
Thus
.
~ Edited by aoum
Solution 3
Using logarithmic identities and the change of base formula, the product can be rewritten as . Then we can separate this into two series.
The latter series is a telescoping series, and it can be pretty easily evaluated to be
. The former can be factored as
, and writing out the first terms could tell us that this is a telescoping series as well. Cancelling out the terms would yield
.
Multiplying the two will give us
, which tells us that the answer is
.
Solution 4 (thorough)
The product is equal to from difference of squares and properties of logarithms. We can now expand:
Thus the answer is
. ~eevee9406
See also
2025 AIME II (Problems • Answer Key • Resources) | ||
Preceded by Problem 3 |
Followed by Problem 5 | |
1 • 2 • 3 • 4 • 5 • 6 • 7 • 8 • 9 • 10 • 11 • 12 • 13 • 14 • 15 | ||
All AIME Problems and Solutions |
The problems on this page are copyrighted by the Mathematical Association of America's American Mathematics Competitions.